PT33 S2 question #19 RC Forum

Prepare for the LSAT or discuss it with others in this forum.
Post Reply
jim-green

Silver
Posts: 804
Joined: Sat Feb 12, 2011 7:55 pm

PT33 S2 question #19 RC

Post by jim-green » Fri Apr 01, 2011 11:19 am

I'm having trouble with PT33 S2 question #19. This question is pretty tricky. A is correct because lines 17-18 say so. However, D also appears correct because of line 21. Any thoughts on why answer D is incorrect? Thanks for your help.

privatemf

New
Posts: 42
Joined: Sun Feb 13, 2011 1:15 am

Re: PT33 S2 question #19 RC

Post by privatemf » Fri Apr 01, 2011 12:35 pm

19. (A) Inference
The major element of ¶ 1’s rosy hypothesis that the
author believes to be correct, as confirmed by lines
16–18, is that through increased CO2 plants’ rate of
photosynthesis could be “enhanced,” or increased,
and thus enhance plant growth “generally.” That’s what
(A) asserts (and what wrong choice (D) contradicts, as
a matter of fact). But (B) is wrong because based on ¶
2 it looks as if “important crops” like corn and
sugarcane will be negatively impacted, while weeds will
be enhanced. (C) gets the author’s main idea 180
degrees wrong: That’s the position he believes to be
incorrect. (E), finally, refers to an element of ¶ 1 (lines
11–13) that the author seems to agree with, not dispute.

According to Kaplan.

jim-green

Silver
Posts: 804
Joined: Sat Feb 12, 2011 7:55 pm

Re: PT33 S2 question #19 RC

Post by jim-green » Fri Apr 01, 2011 1:27 pm

privatemf, thanks! that helps very much.

User avatar
soj

Platinum
Posts: 7888
Joined: Sat Jan 16, 2010 11:10 pm

Re: PT33 S2 question #19 RC

Post by soj » Mon Apr 04, 2011 10:19 am

I know this is kind of late, but here's my explanation, having just done this PT 20 minutes ago:

(D) is wrong because the author WOULD be correct in saying enhanced plant growth could lead to abundances of certain species of plants (line 19-20). While line 21 says it would reduce agricultural yields, it doesn't refer to plants that are unrelated (or even detrimental) to agricultural yields, such as weeds. The passage does support that enhanced plant growth would lead to enhanced growth in weeds and photosynthetic-inefficient plants, so (D) is incorrect.

jim-green

Silver
Posts: 804
Joined: Sat Feb 12, 2011 7:55 pm

Re: PT33 S2 question #19 RC

Post by jim-green » Mon Apr 04, 2011 10:47 am

soj wrote:The passage does support that enhanced plant growth would lead to enhanced growth in weeds and photosynthetic-inefficient plants, so (D) is incorrect.
Thanks, soj.

Want to continue reading?

Register now to search topics and post comments!

Absolutely FREE!


Post Reply

Return to “LSAT Prep and Discussion Forum”